Mathcenter Forum  

Go Back   Mathcenter Forum > คณิตศาสตร์โอลิมปิก และอุดมศึกษา > ข้อสอบโอลิมปิก
สมัครสมาชิก คู่มือการใช้ รายชื่อสมาชิก ปฏิทิน ค้นหา ข้อความวันนี้ ทำเครื่องหมายอ่านทุกห้องแล้ว

ตั้งหัวข้อใหม่ Reply
 
เครื่องมือของหัวข้อ ค้นหาในหัวข้อนี้
  #1  
Old 14 ตุลาคม 2014, 23:06
~!!Arale!!~'s Avatar
~!!Arale!!~ ~!!Arale!!~ ไม่อยู่ในระบบ
เริ่มฝึกวรยุทธ์
 
วันที่สมัครสมาชิก: 01 กรกฎาคม 2014
ข้อความ: 11
~!!Arale!!~ is on a distinguished road
Default สอวน. ม.เกษตรศาสตร์ ค่าย1/2557 สอบครั้งที่1

ศูนย์เกษตรศาสตร์จะแบ่งสอบออกเป็น2รอบ มีกลางค่ายและก็ปลายค่าย
รอบนี้สอบ วิชาตรรกศาสตร์และการพิสูจน์ วิชาทฤษฎีจำนวน และวิชาคอมบินาทอริกค่ะ


ตรรกศาสตร์และการพิสูจน์

1. ให้ a,b เป็นจำนวนเต็ม
จงพิสูจน์ว่า ถ้าab เป็นจำนวนคู่ แล้ว a หรือb จะเป็นจำนวนเต็มคู่

2. ให้ $a_{n}=a_{n-1}-11a_{n-2}+6a_{n-3}$
และ $a_{0}=2,a_{1}=5,a_2=15$
จงแสดงว่า $a_{n}=1-2^{n}+2(3^{n})$ ทุก nเป็นจำนวนนับที่มากกว่าหรือเท่ากับ $3$
โดยใช้หลักอุปนัยเชิงคณิตศาสตร์

3.ให้ $\{f=(x,y)\in\mathbb{R}\times\mathbb{R}\ |\ x^{2}y+4y-x=0 \}$
จงพิสูจน์ว่า 3.1 $D_{f}=\mathbb{R}$
3.2 $R_{f}=[-\frac{1}{4},\frac{1}{4}]$

4. ให้ $f: \mathbb{R}\rightarrow\mathbb{R}\times\mathbb{R}$ และ $f(x)=(5x,x-3)$
4.1 $f$ เป็นฟังก์ชัน1-1 ถ้าจริงให้พิสูจน์ ถ้าไม่จริงให้ยกตัวอย่างค้าน
4.2 $f$ เป็นฟังก์ชันทั่วถึง ถ้าจริงให้พิสูจน์ ถ้าไม่จริงให้ยกตัวอย่างค้าน

5. ให้ $A\subseteq\mathbb{R}$ โดยที่ $A\not=\phi\ และ\ \ell\ เป็นขอบเขตล่างของ A$
จงแสดงว่า $\ell\ เป็นขอบเขตล่างที่มากที่สุดของ A\leftrightarrow จำนวนจริง\ b\ ใดๆ ถ้า\ b>1$
แล้วจะมี $a\in A \ ซึ่ง\ a<b$


ทฤษฎีจำนวน

1. จงแสดงว่า ไม่สามารถแบ่งจำนวน $1,2,3,...,15\ ออกเป็นเซตA\ และเซตB\ ได้ โดยให้$
$A\cup B=\{1,2,3,...,15\},A\cap B=\phi ,|A|=13,|B|=2\ และผลรวมของสมาชิกในเซตA$
$เท่ากับผลคูณของสมาชิกในเซตB$

2. กำหนดให้ $418x+165y=2557+k$
จงหา$k$ ที่เป็นจำนวนนับที่น้อยที่สุดที่ทำให้มี $x,y\in\mathbb{Z}$ สอดคล้อง
กับสมการนี้ และจงแสดงด้วยว่ามีผลเฉลย $x,y$ เพียงคู่เดียวที่เป็นบวกพร้อมกัน

3. จงแสดงว่า ถ้า $a\in\mathbb{N} ,a\geq2$ แล้ว
$(a^{m}-1,a^{n}-1)=a^{(m,n)}-1$

4.ให้ $a,n\in\mathbb{N} และ\ a\geq 2,\ n\geq2\ ซึ่ง\ a^{n}-1$ เป็นจำนวนเฉพาะ
จงพิสูจน์ว่า $a$ จะมีค่าเท่ากับ$\ 2$ และ$\ n$ เป็นจำนวนเฉพาะ


คอมบินาทอริก

1.มีเส้นตรง $x=1,y=1,x=2,y=2,...,x=n,y=n$ อยู่บนระนาบ
จงหาจำนวนสี่เหลี่ยมจัตุรัสที่เกิดจากเส้นตรงตัดกัน

2. สมมติว่าชั้นเรียนหนึ่งมีนักเรียนชาย $n$ คน นักเรียนหญิง $n$ คน
จงสร้างสถานการณ์ โดยใช้การพิสูจน์เชิงการจัด
2.1 สร้างเหตุการณ์ได้ $\left(\matrix{n\\0}\right)^{2}+\left(\matrix{n\\1}\right)^{2}+...+\left(\matrix{n\\n}\right)^{2}$
2.2 จงพิสูจน์ว่า เท่ากับ$\left(\matrix{2n\\n}\right)$

3. ให้ $k=0,1,2,...,2014$
และ $a_{k}=\left(\matrix{2014\\k}\right) \frac{1}{4^{k}}$
จงหาค่า$\ k$ ทุกจำนวน ที่ทำให้ $a_{k}$ มีค่าสูงที่สุด
__________________
You are in control of your destiny,

Only you can make your dreams come true.
ตอบพร้อมอ้างอิงข้อความนี้
  #2  
Old 15 ตุลาคม 2014, 20:01
pond27216's Avatar
pond27216 pond27216 ไม่อยู่ในระบบ
จอมยุทธ์หน้าใหม่
 
วันที่สมัครสมาชิก: 09 ธันวาคม 2013
ข้อความ: 87
pond27216 is on a distinguished road
Default

อ้างอิง:
ข้อความเดิมเขียนโดยคุณ ~!!Arale!!~ View Post
ศูนย์เกษตรศาสตร์จะแบ่งสอบออกเป็น2รอบ มีกลางค่ายและก็ปลายค่าย
รอบนี้สอบ วิชาตรรกศาสตร์และการพิสูจน์ วิชาทฤษฎีจำนวน และวิชาคอมบินาทอริกค่ะ


ตรรกศาสตร์และการพิสูจน์

1. ให้ a,b เป็นจำนวนเต็ม
จงพิสูจน์ว่า ถ้าab เป็นจำนวนคู่ แล้ว a หรือb จะเป็นจำนวนเต็มคู่

2. ให้ $a_{n}=a_{n-1}-11a_{n-2}+6a_{n-3}$
และ $a_{0}=2,a_{1}=5,a_2=15$
จงแสดงว่า $a_{n}=1-2^{n}+2(3^{n})$ ทุก nเป็นจำนวนนับที่มากกว่าหรือเท่ากับ $3$
โดยใช้หลักอุปนัยเชิงคณิตศาสตร์

3.ให้ $\{f=(x,y)\in\mathbb{R}\times\mathbb{R}\ |\ x^{2}y+4y-x=0 \}$
จงพิสูจน์ว่า 3.1 $D_{f}=\mathbb{R}$
3.2 $R_{f}=[-\frac{1}{4},\frac{1}{4}]$

4. ให้ $f: \mathbb{R}\rightarrow\mathbb{R}\times\mathbb{R}$ และ $f(x)=(5x,x-3)$
4.1 $f$ เป็นฟังก์ชัน1-1 ถ้าจริงให้พิสูจน์ ถ้าไม่จริงให้ยกตัวอย่างค้าน
4.2 $f$ เป็นฟังก์ชันทั่วถึง ถ้าจริงให้พิสูจน์ ถ้าไม่จริงให้ยกตัวอย่างค้าน

5. ให้ $A\subseteq\mathbb{R}$ โดยที่ $A\not=\phi\ และ\ \ell\ เป็นขอบเขตล่างของ A$
จงแสดงว่า $\ell\ เป็นขอบเขตล่างที่มากที่สุดของ A\leftrightarrow จำนวนจริง\ b\ ใดๆ ถ้า\ b>1$
แล้วจะมี $a\in A \ ซึ่ง\ a<b$


ทฤษฎีจำนวน

1. จงแสดงว่า ไม่สามารถแบ่งจำนวน $1,2,3,...,15\ ออกเป็นเซตA\ และเซตB\ ได้ โดยให้$
$A\cup B=\{1,2,3,...,15\},A\cap B=\phi ,|A|=13,|B|=2\ และผลรวมของสมาชิกในเซตA$
$เท่ากับผลคูณของสมาชิกในเซตB$

2. กำหนดให้ $418x+165y=2557+k$
จงหา$k$ ที่เป็นจำนวนนับที่น้อยที่สุดที่ทำให้มี $x,y\in\mathbb{Z}$ สอดคล้อง
กับสมการนี้ และจงแสดงด้วยว่ามีผลเฉลย $x,y$ เพียงคู่เดียวที่เป็นบวกพร้อมกัน

3. จงแสดงว่า ถ้า $a\in\mathbb{N} ,a\geq2$ แล้ว
$(a^{m}-1,a^{n}-1)=a^{(m,n)}-1$

4.ให้ $a,n\in\mathbb{N} และ\ a\geq 2,\ n\geq2\ ซึ่ง\ a^{n}-1$ เป็นจำนวนเฉพาะ
จงพิสูจน์ว่า $a$ จะมีค่าเท่ากับ$\ 2$ และ$\ n$ เป็นจำนวนเฉพาะ


คอมบินาทอริก

1.มีเส้นตรง $x=1,y=1,x=2,y=2,...,x=n,y=n$ อยู่บนระนาบ
จงหาจำนวนสี่เหลี่ยมจัตุรัสที่เกิดจากเส้นตรงตัดกัน

2. สมมติว่าชั้นเรียนหนึ่งมีนักเรียนชาย $n$ คน นักเรียนหญิง $n$ คน
จงสร้างสถานการณ์ โดยใช้การพิสูจน์เชิงการจัด
2.1 สร้างเหตุการณ์ได้ $\left(\matrix{n\\0}\right)^{2}+\left(\matrix{n\\1}\right)^{2}+...+\left(\matrix{n\\n}\right)^{2}$
2.2 จงพิสูจน์ว่า เท่ากับ$\left(\matrix{2n\\n}\right)$

3. ให้ $k=0,1,2,...,2014$
และ $a_{k}=\left(\matrix{2014\\k}\right) \frac{1}{4^{k}}$
จงหาค่า$\ k$ ทุกจำนวน ที่ทำให้ $a_{k}$ มีค่าสูงที่สุด
ผมสอบ มก เหมือนกัน แต่อยู่ศูนย์ย่อย ข้่อสุดท้ายคอมบิคิดยังไงเนี่ยครับ -0-
__________________
ทำโจทย์ข้อละ2วัน
ตอบพร้อมอ้างอิงข้อความนี้
  #3  
Old 15 ตุลาคม 2014, 23:04
polsk133's Avatar
polsk133 polsk133 ไม่อยู่ในระบบ
กระบี่ไร้สภาพ
 
วันที่สมัครสมาชิก: 14 สิงหาคม 2011
ข้อความ: 1,873
polsk133 is on a distinguished road
Default

1. B={x,y} then 120-x-y=xy
2. (418,330)|2557+k
3 ยูคลิด
4 แยกตัวประกอบ

1.(n-1)(n)(2n-1)/6
2.-
3. 402,403
ตอบพร้อมอ้างอิงข้อความนี้
  #4  
Old 16 ตุลาคม 2014, 19:10
~!!Arale!!~'s Avatar
~!!Arale!!~ ~!!Arale!!~ ไม่อยู่ในระบบ
เริ่มฝึกวรยุทธ์
 
วันที่สมัครสมาชิก: 01 กรกฎาคม 2014
ข้อความ: 11
~!!Arale!!~ is on a distinguished road
Default

อ้างอิง:
ข้อความเดิมเขียนโดยคุณ pond27216 View Post
ผมสอบ มก เหมือนกัน แต่อยู่ศูนย์ย่อย ข้่อสุดท้ายคอมบิคิดยังไงเนี่ยครับ -0-
เราใช้เป็นฟังก์ชันเพิ่ม-ลดอ่าค่ะ คิดจุดที่เพิ่มสูงสุดละก็คิดจุดที่ค่าเริ่มลดลง ได้ 402,403 เหมือนคุณ polsk133
__________________
You are in control of your destiny,

Only you can make your dreams come true.
ตอบพร้อมอ้างอิงข้อความนี้
  #5  
Old 22 มกราคม 2015, 06:11
pond27216's Avatar
pond27216 pond27216 ไม่อยู่ในระบบ
จอมยุทธ์หน้าใหม่
 
วันที่สมัครสมาชิก: 09 ธันวาคม 2013
ข้อความ: 87
pond27216 is on a distinguished road
Default

อ้างอิง:
ข้อความเดิมเขียนโดยคุณ ~!!Arale!!~ View Post
ศูนย์เกษตรศาสตร์จะแบ่งสอบออกเป็น2รอบ มีกลางค่ายและก็ปลายค่าย
รอบนี้สอบ วิชาตรรกศาสตร์และการพิสูจน์ วิชาทฤษฎีจำนวน และวิชาคอมบินาทอริกค่ะ


ตรรกศาสตร์และการพิสูจน์

1. ให้ a,b เป็นจำนวนเต็ม
จงพิสูจน์ว่า ถ้าab เป็นจำนวนคู่ แล้ว a หรือb จะเป็นจำนวนเต็มคู่

2. ให้ $a_{n}=a_{n-1}-11a_{n-2}+6a_{n-3}$
และ $a_{0}=2,a_{1}=5,a_2=15$
จงแสดงว่า $a_{n}=1-2^{n}+2(3^{n})$ ทุก nเป็นจำนวนนับที่มากกว่าหรือเท่ากับ $3$
โดยใช้หลักอุปนัยเชิงคณิตศาสตร์

3.ให้ $\{f=(x,y)\in\mathbb{R}\times\mathbb{R}\ |\ x^{2}y+4y-x=0 \}$
จงพิสูจน์ว่า 3.1 $D_{f}=\mathbb{R}$
3.2 $R_{f}=[-\frac{1}{4},\frac{1}{4}]$

4. ให้ $f: \mathbb{R}\rightarrow\mathbb{R}\times\mathbb{R}$ และ $f(x)=(5x,x-3)$
4.1 $f$ เป็นฟังก์ชัน1-1 ถ้าจริงให้พิสูจน์ ถ้าไม่จริงให้ยกตัวอย่างค้าน
4.2 $f$ เป็นฟังก์ชันทั่วถึง ถ้าจริงให้พิสูจน์ ถ้าไม่จริงให้ยกตัวอย่างค้าน

5. ให้ $A\subseteq\mathbb{R}$ โดยที่ $A\not=\phi\ และ\ \ell\ เป็นขอบเขตล่างของ A$
จงแสดงว่า $\ell\ เป็นขอบเขตล่างที่มากที่สุดของ A\leftrightarrow จำนวนจริง\ b\ ใดๆ ถ้า\ b>1$
แล้วจะมี $a\in A \ ซึ่ง\ a<b$


ทฤษฎีจำนวน

1. จงแสดงว่า ไม่สามารถแบ่งจำนวน $1,2,3,...,15\ ออกเป็นเซตA\ และเซตB\ ได้ โดยให้$
$A\cup B=\{1,2,3,...,15\},A\cap B=\phi ,|A|=13,|B|=2\ และผลรวมของสมาชิกในเซตA$
$เท่ากับผลคูณของสมาชิกในเซตB$

2. กำหนดให้ $418x+165y=2557+k$
จงหา$k$ ที่เป็นจำนวนนับที่น้อยที่สุดที่ทำให้มี $x,y\in\mathbb{Z}$ สอดคล้อง
กับสมการนี้ และจงแสดงด้วยว่ามีผลเฉลย $x,y$ เพียงคู่เดียวที่เป็นบวกพร้อมกัน

3. จงแสดงว่า ถ้า $a\in\mathbb{N} ,a\geq2$ แล้ว
$(a^{m}-1,a^{n}-1)=a^{(m,n)}-1$

4.ให้ $a,n\in\mathbb{N} และ\ a\geq 2,\ n\geq2\ ซึ่ง\ a^{n}-1$ เป็นจำนวนเฉพาะ
จงพิสูจน์ว่า $a$ จะมีค่าเท่ากับ$\ 2$ และ$\ n$ เป็นจำนวนเฉพาะ


คอมบินาทอริก

1.มีเส้นตรง $x=1,y=1,x=2,y=2,...,x=n,y=n$ อยู่บนระนาบ
จงหาจำนวนสี่เหลี่ยมจัตุรัสที่เกิดจากเส้นตรงตัดกัน

2. สมมติว่าชั้นเรียนหนึ่งมีนักเรียนชาย $n$ คน นักเรียนหญิง $n$ คน
จงสร้างสถานการณ์ โดยใช้การพิสูจน์เชิงการจัด
2.1 สร้างเหตุการณ์ได้ $\left(\matrix{n\\0}\right)^{2}+\left(\matrix{n\\1}\right)^{2}+...+\left(\matrix{n\\n}\right)^{2}$
2.2 จงพิสูจน์ว่า เท่ากับ$\left(\matrix{2n\\n}\right)$

3. ให้ $k=0,1,2,...,2014$
และ $a_{k}=\left(\matrix{2014\\k}\right) \frac{1}{4^{k}}$
จงหาค่า$\ k$ ทุกจำนวน ที่ทำให้ $a_{k}$ มีค่าสูงที่สุด
ขุดๆ ช่วยเฉลยคอมบิข้อแรกหน่อยครับ -?.-
__________________
ทำโจทย์ข้อละ2วัน
ตอบพร้อมอ้างอิงข้อความนี้
  #6  
Old 22 มกราคม 2015, 22:03
Beatmania's Avatar
Beatmania Beatmania ไม่อยู่ในระบบ
ลมปราณคุ้มครองร่าง
 
วันที่สมัครสมาชิก: 10 พฤษภาคม 2011
ข้อความ: 279
Beatmania is on a distinguished road
Default

รูปที่ได้จะเป็นสี่เหลี่ยมจัตุรัสขนาด $(n-1)\times (n-1)$

เราจะนับจำสี่เหลี่ยมจัตุรัสที่มีความยาวด้าน $k$

เราเห็นว่า สี่เหลี่ยมจัตุรัสที่มีความยาวด้าน $k$ รูปหนึ่งเกิดจากคู่ของเส้นตรงที่ขนานแกน $X,Y$ $2$ คู่โดยแต่ละคู่มีระยะห่าง $k$

เราเลือกเส้นตรงที่ขนานแกน $X$ และมีระยะห่างกัน $k$ ได้ $n-k$ คู่

เราเลือกเส้นตรงที่ขนานแกน $Y$ และมีระยะห่างกัน $k$ ได้ $n-k$ คู่

แสดงว่า จำนวนของรูปสี่เหลี่ยมจัตุรัสความยาวด้าน $k$ จึงเท่ากับ $(n-k)^2$

เรานับจำนวนของรูปสี่เหลี่ยมจัตุรัสทั้งหมด เราสามารถนับได้โดย run $k=1,2,...,n-1$

ทำให้ได้จำนวนของรูปสี่เหลี่ยมเท่ากับ $1^2+2^2+...+(n-1)^2=\frac{(n-1)n(2n-1)}{6}$ รูปครับ
__________________
I'm Back
ตอบพร้อมอ้างอิงข้อความนี้
  #7  
Old 16 เมษายน 2015, 16:49
Pitchayut Pitchayut ไม่อยู่ในระบบ
บัณฑิตฟ้า
 
วันที่สมัครสมาชิก: 20 มกราคม 2015
ข้อความ: 352
Pitchayut is on a distinguished road
Default

Combi ข้อ 3 ถ้าเริ่มต้นถูกก็คือจบครับ

$a_k$ จะมีค่ามากที่สุดก็ต่อเมื่อ $k$ เป็นจำนวนที่มีค่าน้อยที่สุดที่ทำให้ $\dfrac{a_k}{a_{k+1}}\le1$ (จริงไหมครับ)

ดังนั้น จะได้

$$\dfrac{\displaystyle{\binom{2014}{k}}}{4^k}\le \dfrac{\displaystyle{\binom{2014}{k+1}}}{4^{k+1}}$$

เพราะว่า
$$\binom{2014}{k}=\frac{2014!}{(2014-k)!k!}=\frac{k+1}{2014-k}\cdot\frac{2014!}{(2013-k)!(k+1)!}=\frac{k+1}{2014-k}\binom{2014}{k+1}$$
เมื่อนำไปแทนในอสมการดังกล่าวและจัดรูป จะได้
$$\frac{4k+4}{2014-k}\ge 1$$
เมื่อแก้อสมการ (D.I.Y) จะได้
$$k\ge 402$$
แต่ว่า $a_{402}=a_{403}$ (จากอสมการที่แก้)
นั่นคือ $k$ ที่ทำให้ $a_k$ มีค่ามากที่สุดคือ $402$ และ $403$

05 พฤษภาคม 2015 17:53 : ข้อความนี้ถูกแก้ไขแล้ว 2 ครั้ง, ครั้งล่าสุดโดยคุณ Pitchayut
เหตุผล: แก้ Latex code
ตอบพร้อมอ้างอิงข้อความนี้
ตั้งหัวข้อใหม่ Reply


หัวข้อคล้ายคลึงกัน
หัวข้อ ผู้ตั้งหัวข้อ ห้อง คำตอบ ข้อความล่าสุด
ข้อสอบ สพฐ. 2557 รอบที่ 2 (9 มี.ค.57) gon ข้อสอบในโรงเรียน ประถมปลาย 27 11 มีนาคม 2015 17:30
ฤดูกาลแข่งขันคณิตศาสตร์ 2557 เริ่มแล้ว เสือน้อย ข่าวคราวแวดวง ม.ต้น 1 27 สิงหาคม 2014 22:27
ฤดูแข่งขันคณิตศาสตร์ปี 2557 เริ่มแล้ว เสือน้อย ข่าวคราวแวดวงประถม ปลาย 0 22 พฤษภาคม 2014 10:27
สพฐ. 2557 กำหนดการรับสมัคร(1-25 ธ.ค.2556)และสอบแข่ง รอบที่ 1 (26 ม.ค.2557) gon ข่าวคราวแวดวง ม.ต้น 22 16 ธันวาคม 2013 09:56
สพฐ. 2557 กำหนดการรับสมัคร(1-25 ธ.ค.2556)และสอบแข่ง รอบที่ 1 (26 ม.ค.2557) gon ข่าวคราวแวดวงประถม ปลาย 1 10 พฤศจิกายน 2013 04:56

เครื่องมือของหัวข้อ ค้นหาในหัวข้อนี้
ค้นหาในหัวข้อนี้:

ค้นหาขั้นสูง

กฎการส่งข้อความ
คุณ ไม่สามารถ ตั้งหัวข้อใหม่ได้
คุณ ไม่สามารถ ตอบหัวข้อได้
คุณ ไม่สามารถ แนบไฟล์และเอกสารได้
คุณ ไม่สามารถ แก้ไขข้อความของคุณเองได้

vB code is On
Smilies are On
[IMG] code is On
HTML code is Off
ทางลัดสู่ห้อง


เวลาที่แสดงทั้งหมด เป็นเวลาที่ประเทศไทย (GMT +7) ขณะนี้เป็นเวลา 17:33


Powered by vBulletin® Copyright ©2000 - 2024, Jelsoft Enterprises Ltd.
Modified by Jetsada Karnpracha